subject
Mathematics, 21.03.2020 10:04 Jackie2088

A magazine advertisement claims that wearing a magnetized bracelet will reduce arthritis pain in those who suffer from arthritis. A medical researcher tests this claim with 233 arthritis sufferers randomly assigned to wear either a magnetized bracelet or a placebo bracelet. The researcher records the proportion of each group who report relief from arthritis pain after 6 weeks. After analyzing the data, he fails to reject the null hypothesis. What are valid interpretations of his findings? Which of the following are valid interpretations of his findings? There may be more than one correct answer. Select all answers that apply A. The magnetized bracelets had exactly the same effect as the placebo at reducing arthritis pain. B. The magnetized bracelets are not effective at reducing arthritis pain C. There were no statistically significant differences between the magnetized bracelets and the placebos in reducing arthritis pain. D. There's insufficient evidence that the magnetized bracelets are effective at reducing arthritis pain.

ansver
Answers: 3

Another question on Mathematics

question
Mathematics, 21.06.2019 19:10
Revirite the formula to determine the area covered by one can of paint. alyssa and her father are going to paint the house this summer. they know they'll need 6 cans of paint. enter the correct answer. 000 done a formula that describes this scenario is (licierali 000 oo 6 w w = the total area of the walls p = area covered by a can of paint
Answers: 1
question
Mathematics, 21.06.2019 20:40
If the endpoints of the diameter of a circle are (8, 6) and (2,0), what is the standard form equation of the circle? a) (x + 5)2 + (y + 3)2 = 18 (x + 5)2 + (y + 3)2 = 3.72 (x - 5)2 + (y - 3)2 = 18 d) (x - 5)2 + (y - 3)2 = 32
Answers: 1
question
Mathematics, 21.06.2019 21:30
Which best describes the construction of a triangle if given the segment lengths of 2 cm, 3 cm, and 5 cm? a) unique triangle b) cannot be determined c) triangle not possible d) more than one triangle
Answers: 1
question
Mathematics, 21.06.2019 23:00
Will give a: 122 b: 90 c: 48 d: 180
Answers: 1
You know the right answer?
A magazine advertisement claims that wearing a magnetized bracelet will reduce arthritis pain in tho...
Questions
question
Chemistry, 06.07.2019 15:00
question
Arts, 06.07.2019 15:00
question
Mathematics, 06.07.2019 15:00
question
Health, 06.07.2019 15:00
question
Mathematics, 06.07.2019 15:00
Questions on the website: 13722367